0 Daumen
649 Aufrufe

Folgende Reihe ist gegeben:

$$\sum _{ n=1 }^{ \infty  }{ \cfrac { 1-n-\frac { 1 }{ \sqrt { n }  }  }{ 3\times { n }^{ 3 }-2+n }  } $$

Ich kenne alle Konvergenzkriterien.

Das notwenidge Kriterium trifft schonmal zu. Ich vermute, dass die Teleskopreihe hier irgendwie versteckt ist.

Zuerst habe ich versucht den Bruch auseinander zu ziehen, um zu sehen, ob es zumindest eine Ähnlichkeit mit der Teleskopreihe haben könnte.

$${ \cfrac { 1-n-\frac { 1 }{ \sqrt { n }  }  }{ 3\times { n }^{ 3 }-2+n }  }\quad \\ =\quad \cfrac { 1 }{ 3\ast { n }^{ 3 }-2+n } -\cfrac { n }{ 3\ast { n }^{ 3 }-2+n } -\cfrac { \frac { 1 }{ \sqrt { n }  }  }{ 3\ast { n }^{ 3 }-2+n } \\ =\quad \frac { 1 }{ 3\ast { n }^{ 3 }-2+n } -\cfrac { n }{ 3\ast { n }^{ 3 }-2+n } -\cfrac { 1 }{ \sqrt { n } \ast (3\ast { n }^{ 3 }-2+n) }  $$


Ob das nun zu viel Fantasie ist da die Teleskopreihe zu sehen, weiß ich nicht.

Es wäre aber schön es zu wissen.

Meine zweite Idee war, diese Summe in drei Summen aufzuteilen und diese einzelnd zu betrachten und dann die differenz zu ziehen. Dies geht aber nur wenn die Reihe konvergent ist. Und wenn ich das vorher weiß, dann bin ich ja theoretisch schon fertig.

Ich bin froh für jeden Tipp und bitte darum, dass es bei einem Tipp bleibt. Ich würde dies schon gerne selbst ausrechnen, habe nur das Gefühl etwas auf der Stelle zu treten.

:)

Avatar von

1 Antwort

+1 Daumen
 
Beste Antwort
Vielleicht mit dem Majorantenkriterium. Du hast den Bruch ja auseinander gezogen.
Ich würde jetzt den ersten Teil(den 1. Summand) abschätzen mit 1/n^3 und die Minuenden weg lassen,damit es noch größer wird.
Also ist das ganze kleiner als ∑1/n^3  . Das konvergiert, also konvergiert auch deine Reihe
Und das abschätzen würde ich machen da für n = 2 schon Gleichheit zwischen dem ersten Teil und 1/n^3 gilt. Und danach ists auf jeden Fall kleiner als <3
Der Nenner ist außerdem nie Null für n<=1 .
Also denke ich mal das man bei der Abschätzung sagen kann, für genügen großes n ( n=2 ) ist die Reihe kleiner.
Ich wüsste nicht,wie ich dir sonst nur einen Tipp geben könnte.

Ich garantiere aber nicht für Richtigkeit.

Was aufjedenfall stimmt, ist,dass die Reihe konvergiert.
Avatar von 8,7 k
Geht das so in etwa durch (habe es extra sehr kleinschrittig gemacht, zum Verständnis):

$$\left| { \cfrac { 1-n-\frac { 1 }{ \sqrt { n }  }  }{ 3\times { n }^{ 3 }-2+n }  }\quad  \right| \quad \\ =\quad \left| \cfrac { 1 }{ 3\ast { n }^{ 3 }-2+n } -\cfrac { n }{ 3\ast { n }^{ 3 }-2+n } -\cfrac { \frac { 1 }{ \sqrt { n }  }  }{ 3\ast { n }^{ 3 }-2+n }  \right| \\ =\quad \left| \frac { 1 }{ 3\ast { n }^{ 3 }-2+n } -\cfrac { n }{ 3\ast { n }^{ 3 }-2+n } -\cfrac { 1 }{ \sqrt { n } \ast (3\ast { n }^{ 3 }-2+n) }  \right| \\ =\quad \left| \frac { 1 }{ 3\ast { n }^{ 3 }-2+n } +(-\cfrac { n }{ 3\ast { n }^{ 3 }-2+n } )+(-\cfrac { 1 }{ \sqrt { n } \ast (3\ast { n }^{ 3 }-2+n) } ) \right| \\ \le \left| \frac { 1 }{ 3\ast { n }^{ 3 }-2+n }  \right| +\left| (-\cfrac { n }{ 3\ast { n }^{ 3 }-2+n } ) \right| +\left| (-\cfrac { 1 }{ \sqrt { n } \ast (3\ast { n }^{ 3 }-2+n) } ) \right| \\ <\quad \frac { 1 }{ { n }^{ 3 } } \quad +\quad \frac { n }{ { n }^{ 3 } } \quad +\quad \frac { 1 }{ { n }^{ 3 } } \\ =\quad \frac { 1 }{ { n }^{ 3 } } \quad +\quad \frac { 1 }{ { n }^{ 2 } } \quad +\quad \frac { 1 }{ { n }^{ 3 } } \\ <\quad \frac { 3 }{ { n }^{ 2 } } \\ <\frac { 1 }{ { n }^{ 2 } }  $$

Sei nun |an| = $$\left| { \cfrac { 1-n-\frac { 1 }{ \sqrt { n }  }  }{ 3\times { n }^{ 3 }-2+n }  }\quad  \right| \quad $$
und bn = $$\frac { 1 }{ { n }^{ 2 } } $$

Da durch |an| ≤ bn , wobei ∑bn konvergent ist, das Majorantenkriterium erfüllt ist, folgt daraus, dass ∑an auch konvergent ist.

Da ∑bn sogar absolut konvergent ist, konvergiert auch ∑an absolut.

Kann mir das jemand absegnen, oder habe ich irgendwo einen kleinen Fehler, bzw. kann man etwas besser machen?

ups 3/n^2 ist nicht kleiner als 1/n^2..

aber 3/n^2 ist auch konvergent... kann ich das dann so lassen? :D


Oder ich schreibe 3* ∑ 1/n^2

Man kann direkt sagen,dass 3/n^2 konvergiert. Und sich bn=3/n^2 wählen.

Ich danke Dir sehr :)

Ein anderes Problem?

Stell deine Frage

Willkommen bei der Mathelounge! Stell deine Frage einfach und kostenlos

x
Made by a lovely community